LSAT and Law School Admissions Forum

Get expert LSAT preparation and law school admissions advice from PowerScore Test Preparation.

User avatar
 Dave Killoran
PowerScore Staff
  • PowerScore Staff
  • Posts: 5853
  • Joined: Mar 25, 2011
|
#41161
Complete Question Explanation
(The complete setup for this game can be found here: lsat/viewtopic.php?t=11758)

The correct answer choice is (C)

G must be in the level 1 class, and thus answer choice (B) is eliminated. F, K, and N appear near the end of the chain, and all can be in the level 3 class, and thus answer choices (A), (D), and (E) are eliminated. As discussed during the setup, J cannot be in the level 3 class, and thus answer choice (C) is correct.

Get the most out of your LSAT Prep Plus subscription.

Analyze and track your performance with our Testing and Analytics Package.